LSAT and Law School Admissions Forum

Get expert LSAT preparation and law school admissions advice from PowerScore Test Preparation.

 Administrator
PowerScore Staff
  • PowerScore Staff
  • Posts: 8917
  • Joined: Feb 02, 2011
|
#26035
Complete Question Explanation
(The complete setup for this game can be found here: lsat/viewtopic.php?t=10807)

The correct answer choice is (A)


This is probably the most difficult question in this game, because an efficient approach is not immediately apparent. When your main setup does not contain the information to answer a Global Cannot Be True question immediately, your first step should be to examine prior work.

In question #14, our local diagram shows that M and R could both be located in aisle 2 without violating any of the rules. This eliminates answer choice (B). For the remaining four answer choices, consider the variables listed and their effect on the setup:

Answer choice (A): This is the correct answer choice. If H and M are the only variables in aisle 2, this would force S to be in aisle 3, which is impossible thanks to the third rule of the game. Therefore, H and M cannot be the sections located in aisle 2.

Answer choice (C): This answer choice seems unlikely to be correct. From question #14, we know that M and R can both be located in aisle 2 without violating any of the rules. The same is also true for M and S. Although S and R are not functionally identical, they function similarly in this context:
oct12_game_3_#16_diagram_1.png
Answer choice (D): This answer choice is also unlikely to be correct. We already know that H and S can form a block in aisle 2, and there is no reason why this block cannot be joined by M:
oct12_game_3_#16_diagram_2.png
Answer choice (E): M, R, and S could all be located in aisle 2, as shown in the setup below, making answer choice (E) also incorrect:
oct12_game_3_#16_diagram_3.png
You do not have the required permissions to view the files attached to this post.
 lina2020
  • Posts: 20
  • Joined: Jul 23, 2020
|
#80138
The question stem confused me, "Which one of the following CANNOT be the list of the sections located in aisle 2?"

I've always seen this question type include "a COMPLETE list of the sections" or similar wording within the stem, but the way this one is written, it does not limit the variables in each aisle to only those stated in each answer choice but rather asks if those two or three variables can be together is aisle 2. Please explain how you can made that distinction.
User avatar
 KelseyWoods
PowerScore Staff
  • PowerScore Staff
  • Posts: 1079
  • Joined: Jun 26, 2013
|
#80169
Hi Lina!

It's good that you're paying careful attention to the wording of question stems like that! Every little word makes a difference and it's always important to know whether they're asking about a complete list or just a partial list.

In this case, the question stem states: "Which one of the following CANNOT be the list of the sections located in aisle 2?" How do we know that they're asking about a complete list? It's subtle--but again it comes down to those small words! Specifically, in this case, it's all about those "the's"!

"Which one of the following CANNOT be the list of the sections located in aisle 2?"

If they're asking about the list of the sections in aisle 2, they're really asking for a complete list here. Would it be clearer if they had included a "complete"? Sure! But there still isn't really another way of interpreting it. If it's the list, consider it the complete list. If they want a partial list, they'll specify a partial list.

Keep up that close reading!

Hope this helps!

Best,
Kelsey
User avatar
 chickfu
  • Posts: 6
  • Joined: Jun 28, 2022
|
#95981
I do not understand why Answer A is correct. Why is S forced into the third aisle? I thought S could also be in the same aisle as H.


Aisle 1: F
Aisle 2: S, M, H
Aisle 3: P

R in Aisles 1 or 2.
User avatar
 chickfu
  • Posts: 6
  • Joined: Jun 28, 2022
|
#95982
chickfu wrote: Tue Jun 28, 2022 11:27 pm I do not understand why Answer A is correct. Why is S forced into the third aisle? I thought S could also be in the same aisle as H.


Aisle 1: F
Aisle 2: S, M, H
Aisle 3: P

R in Aisles 1 or 2.
Oh never mind. I see that the question stem is supposedly indicating that only the variables listed in the answer can be in that aisle.

Get the most out of your LSAT Prep Plus subscription.

Analyze and track your performance with our Testing and Analytics Package.